[Math] Difficulty in Solution of Poisson’s equation using Fourier Transform

calculusfourier transformpartial differential equationspoisson's equation

I wanted to find the potential on an infinite plane due to a point charge located at some point $(x_0,y_0,z_0)$.
So I decided to solve the $2D$ Poisson's equation.
$$\nabla^2 V(x,y)=-\frac{q}{\epsilon}\delta (x-x_0)\delta (y-y_0)$$

Rather than the conventional Green's Function, my method of approach was Fourier transform. Twice application of FT gave
$$V(k,l)=\frac{q}{2\pi \epsilon}\cdot \frac{\exp\left(ikx_0+ily_0\right)}{k^2+l^2}$$

Then I started doing the inverse transforms. After two inverse fourier transforms, I got the integral
$$V(x,y)=\frac{q}{4\pi \epsilon}\cdot \int_{-\infty}^{\infty} \frac{\exp\left(ik(x-x_0)+k\lvert y-y_0\rvert\right)}{k} dk$$

Now this integral on computation using complex analysis gives $\pi i$. And the potential becomes complex. Whereas it should have been logarithmic.

Can someone tell me where I am mistaken? And how I can better compute the integral?

Or maybe the problem lies in the first few steps??

Best Answer

You can use the Fourier Transform theory to seek for a solution. I think you are wrong in making your inverse transforms.

The first task is to transpose your problem into Fourier space. We shall use vector notation letting first $\vec x = (x,y)$, $\vec k=(k_x,k_y)$ the wave vector conjugated in 2D Fourier plane with spatial position $\vec x$ in direct plane. Let's write $\delta(\vec x-\vec x_0)$ the 2D delta distribution in direct plane. We define the direct and inverse Fourier transform of a distribution of density $f(x)$ by,

$$\hat f(\vec k)=\int f(\vec x) \exp(-i \vec k.\vec x) d\vec x$$

and its inverse (normalization of the FT is inessential in what follows),

$$f(x)=\int \hat{f}(\vec k) \exp(i \vec k.\vec x) d\vec k$$

You seek for solving the Poisson equation,

$$\nabla^2 v(\vec x) = -\frac{q}{\epsilon} \delta(\vec x-\vec x_0)$$

It is very easy to show that the action of the Laplacian operator $\nabla^2 v(\vec x)$ translates on Fourier plane into a multiplication of $\hat v(\vec k)$ with the opposite of the squared modulus of the wave vector $-k^2$. Transforming the RHS of the equation is also straightforward when using the basic properties of the Dirac function. Hence your equation translates on Fourier plane into, $$\hat v(\vec k)=\frac{q}{\epsilon} \frac{1}{k^2} \exp(-i \vec k.\vec x_0)$$

Take the inverse Fourier transform of the preceding relation side by side,

$$ v(\vec x, \vec x_0) = \frac{q}{\epsilon} \int \frac{1}{k^2} \exp[-i \vec k.(\vec x-\vec x_0)] d\vec k$$

Make use of a system of polar coordinates on the Fourier plane writing the wave vector $\vec k = k \hat{e}_k(\theta)$ with $\hat e_k(\theta)$ the local radial unit vector of cartesian coordinates $(\cos\theta,\sin\theta)$ on the Fourier plane, where $\theta$ is the polar angle. Using this new system of coordinates, the inverse Fourier transforms into,

$$ v(\vec x, \vec x_0) = \frac{q}{\epsilon} \int_{0}^{\infty} dk \int_{0}^{2\pi} \frac{1}{k} \exp(-i k \hat e_k(\theta).(\vec x-\vec x_0) d\theta$$

Similarly, make use of a system of polar coordinates on the spatial plane to write down the displacement $(\vec x-\vec x_0)$ as $(\vec x-\vec x_0)=r \hat e_r(\phi)$ with unit radial vector $\hat e_r(\phi)=(\cos \phi,\sin\phi)$ where $\phi$ is the polar angle on the spatial plane. This yields,

$$v(\vec x,\vec x_0) \equiv v(r=|\vec x-\vec x_0|,\phi) = \frac{q}{\epsilon} \int_{0}^{\infty} dk \int_{0}^{2\pi} \frac{1}{k} \exp[-i kr (\hat e_k(\theta).\hat e_r(\phi))] \ d\theta$$

The scalar product $\hat e_k(\theta).\hat e_r(\phi)$ is simply $\cos(\theta-\phi)$. It is always possible to choose the referent cartesian frame of the polar system such that $\phi=0$. Hence the transform is isotropic (does not depend upon $\phi$) reducing to,

$$ v(r) = \frac{q}{\epsilon} \int_{0}^{\infty} dk \int_{0}^{2\pi} \frac{1}{k} \exp[-i kr \cos\theta] \ d\theta$$

Integration over polar angle $\theta$ is straightforward it gives the Bessel function of the first kind $2\pi J_0(kr)$. As a conclusion you obtain an integral representation of your solution.

$$v(r)=\frac{2\pi q}{\epsilon} \int_{0}^{\infty} \frac{ J_0(kr)}{kr} d(kr)$$

Note that this integral representation is divergent. Introducing a strictly positive lower bound $a>0$ in the low part of the wavenumber spectrum allows to recover a definite solution. Further integrating over new variable $s=kr$,

$$v(r,a)=\frac{2\pi q}{\epsilon} \int_{ar}^{\infty} \frac{ J_0(s)}{s} ds$$

For instance mathematica/wolfram gives a function which is equivalent at the vicinity of $a=0$ to,

$$v(r,a) \sim -\frac{2\pi q}{\epsilon} [\log(\frac{1}{2} ar) +\gamma]$$ where $\gamma$ is the Euler-Mascheroni constant.

Related Question